average of 2721 2557 2999 2278 4339

Answers

Answer 1

Answer:

Step-by-step explanation:

So we know that the average of numbers is all of the numbers added up and divided by the total amount of numbers.

      2721

   + 2557

-------------------

       5278

.... AND SO ON.........

=14,894 is all of the number added together!!!

Then we count up the numbers= 5

       14,894/5

=2978.8

I hope this helps!!!


Related Questions

Find X using the Angle Sum Theorem

Answers

Answer: x = 125 degree

Explanation:

In a triangle the sum of their angles is 180 degree

Find x:

25 + 30 + x = 180
55 + x = 180
x = 180 - 55
x = 125

Answer:

Step-by-step explanation:

x + 30 + 25 = 180

x + 55 = 180

x = 125

y + 125 = 180

y = 55

What is the solution set for StartAbsoluteValue z + 4 EndAbsoluteValue greater-than 15? 11 less-than z less-than 19 Negative 19 less than z less-than 11 z less-than negative 19 or z greater-than 11 z less-than 19 or z greater-than 11

Answers

Answer:

z less-than negative 19 or z greater-than 11

Step-by-step explanation:

Given the inequality [tex]|z+4|>15[/tex], we are to find the solution set of the inequality. Since the the function is an absolute value, this means that the function will be positive and negative.

For the positive value of the function;

[tex]z+4>15\\\\subtract\ 4\ from \ both \ sides\\z+4-4 > 15 -4\\\\z>11[/tex]

For the negative value of the function we have;

[tex]-(z+4) > 15\\\\-z-4> 15\\add\ 4 \ to\ both \ sides\\\\-z-4+4> 15+4\\\\-z> 19\\\\[/tex]

Multiplying both sides of the inequality by -1 will change the sense of the inequality sign;'

[tex]-(-z)< -19\\\\z<-19[/tex]

Hence the solution sets are [tex]z> 11 \ and \ z< -19 \\[/tex] OR z less-than negative 19 or z greater-than 11

Answer:

z less-than negative 19 or z greater-than 11

Step-by-step explanation:

Need Help
Please Show Work​

Answers

Answer:

-36

Step-by-step explanation:

3*12=36

she is going down (negative) so, it is -36

not sure if this is what you are asking for, if not try this

0-12-12-12=-36

If A = { 10, 30,} B = { 10, 20, 30, 40, 50, 60, 70, 80,90} find A ∩ B There are options. Choose one option only: A- { 30 ,10} B- { 90 ,30 ,10} C- { 90 } D- { 80, 70, 60, 50, 40, 20 }

Answers

[tex]A \cap B=\{10,30\}[/tex]

Answer:

[tex] \boxed{ \purple{10 \: , 30}}[/tex]

Option A is the correct option

Step-by-step explanation:

[tex] \mathrm{Given}[/tex]

A = { 10 , 30 }

B = { 10 , 20 , 30 , 40 , 50 , 60 , 70 , 80 , 90 }

Now, let's find A ∩ B

A ∩ B = { 10 , 30 } ∩ { 10 , 20 , 30 , 40 , 50 , 60 , 70 , 80 , 90 }

The intersection of sets A and B is the set of all elements which belong to both A and B

A B = { 10 , 30 }

The intersection of sets A and B is denoted by ( A ∩ B ) and read as A intersection B.

Hope I helped!

Best regards!

Please Solve
F/Z=T for Z

Answers

Answer:

F /T = Z

Step-by-step explanation:

F/Z=T

Multiply each side by Z

F/Z *Z=T*Z

F = ZT

Divide each side by T

F /T = ZT/T

F /T = Z

Answer:

[tex]\boxed{\red{ z = \frac{f}{t} }}[/tex]

Step-by-step explanation:

[tex] \frac{f}{z} = t \\ \frac{f}{z} = \frac{t}{1} \\ zt = f \\ \frac{zt}{t} = \frac{f}{t} \\ z = \frac{f}{t} [/tex]

Look at the figure below. which ratio represents tan 0?
A -5/4, B -4/5, C -3/4, D 3/5.

Answers

The correct answer is D) 3/5

The required value of the tanФ is given as -3/4. C option is correct.

What is simplification?

The process in mathematics to operate and interpret the function to make the function or expression simple or more understandable is called simplifying and the process is called simplification.

What are trigonometric equations?

These are the equation that contains trigonometric operators such as sin, cos.. etc. In algebraic operations.

here,
Tan(180 - Ф) = -tanФ = perpendicular / base

From figure,  perpendicular= 12 and  base = 16
-tanФ = 12 / 16
tanФ = -3/4

Thus, the required value of the tanФ is given as -3/4. C option is correct.

Learn more about trigonometry equations here:

brainly.com/question/22624805

#SPJ5


Solve for W.
W/9 = g​

Answers

Answer:

W = 9 * g

Step-by-step explanation:

W/9 = g

W = 9 * g

The expression W/9 = g can be written as W = 9g after cross multiplication.

What is an expression?

It is defined as the combination of constants and variables with mathematical operators.

We have an expression:

W/9 = g

To solve for W

Make subject as W:

W = 9g

By cross multiplication.

Thus, the expression W/9 = g can be written as W = 9g after cross multiplication.

Learn more about the expression here:

brainly.com/question/14083225

#SPJ2

Find the value of f (x)=x²-4 and g(x)=3x+2 Find the value of f (-1)g(-1)

Answers

Answer:

3

Step-by-step explanation:

[tex]f(-1)g(-1) \text{ is the same thing as } f(-1)\cdot g(-1). \\\text{Therefore, find f(-1) and g(-1)}[/tex]

[tex]f(-1)=(-1)^2-4\\f(-1)=1-4\\f(-1)=-3[/tex]

[tex]g(-1)=3(-1)+2\\g(-1)=-3+2\\g(-1)=-1[/tex]

Therefore:

[tex]f(-1)\cdot g(-1)\\=(-3)(-1)=3[/tex]

State the correct polar coordinate for the graph shown.

It is not the option selected.

Answers

Answer: Choice D.  (-2.5, 3pi/2)

One way to write this polar coordinate is to say (2.5, pi/2) meaning we move 2.5 units away from the origin toward the pi/2 direction

pi/2 radians = 90 degrees

An alternative is to write (-2.5, 3pi/2) which is where we aim at the 3pi/2 direction (270 degrees) and walk backward while still facing directly south, and we'll arrive at the same location.

Use mathematical induction to prove the statement is true for all positive integers n. The integer n3 + 2n is divisible by 3 for every positive integer n.

Answers

Answer:

Prove:

Using 1

n³+2n = (1)³+2(1) = 1+2= 3 ---> 3/3= 1 ✔

Using 2

n³+2n = (2)³+2(2)= 8+4=12 --> 12/3=4✔

Using 3

n³+2n= (3)³+2(3)= 27+6= 33 --> 33/3=11✔

So it is proven that n³+2n is divisible by 3 for every positive integer.

I hope this helps

if u have question let me know in comments

Could anyone help me with this question please? Thank you.

Answers

Answer:

  C)  549 km²

Step-by-step explanation:

The area of the regular pentagon is given by ...

  A = (1/2)Pa

where P represents the perimeter, and 'a' represents the apothem (6.2 km). Of course, the perimeter is 5 times the side length.

The lateral area is the product of the perimeter and the height:

  LA = Ph

Using these formulas, and recognizing the total area includes two (2) pentagons, we have ...

  total area = (LA) +2(A) = Ph +2(1/2)Pa = P(h +a)

  = (45 km)(6 km +6.2 km) = 549 km^2

Find interval of increase and decrease of f(x) = 8 sin(x) + cot(x), −π ≤ x ≤ π

Answers

Answer:

Given f(x)=8sin(x)+cot(x) for -pi<x<pi :

Note that:

f'(x)=8cos(x)-csc^2(x)

f''(x)=-8sin(x)+2csc^2(x)cot(x)

(1) To find the intervals where f(x) is increasing or decreasing we use the first derivative test; if the first derivative is positive on an interval the functio is increasing, negative implies the functio is decreasing.

Using technology we find the approximate zeros of f'(x) on -pi<x<pi :

x~~-1.443401

x~~-.3752857

x~~.3752857

x~~1.443401

Plugging in test values on the intervals yields:

f'(x)<0 on (-pi,-1.443401)

f'(x)>0 on (-1.443401,-.3752857)

f'(x)<0 on

Plz correct me if wrong

Is the square root of 65 a rational number

Answers

Answer:

No

Step-by-step explanation:

The square root of 65 is irrational.

It is not a rational number because 65 is not a perfect square.

The square root of 65 is 8.06225775...

The square root of 65 is not a rational number.

65 is not a perfect square which means it's impossible to

find a whole number times itself to give us 65.

On a calculator if you type in the square root of 65,

you will get an infinite decimal number.

The decimal values never end and never have same repeated pattern.

Find the domain and the range of the relation.
Find the domain of the relation. Select the correct choice below and fill in the answer box to
complete your choice.

O A. The domain is _
(Type your answer in interval notation.)
B. The domain is {_}
(Type an integer or a fraction. Use a comma to separate answers as needed.)
Find the range of the relation. Select the correct choice below and fill in the answer box to
complete your choice.
O A. The range is _
(Type an integer or a fraction. Use a comma to separate answers as needed.)
OB. The range is {_}

Answers

Answer:

1) the domain is all real numbers

2) the range is

[tex]y \geqslant 3[/tex]

Solve 2x+2y=6 and 3x-2y=11

Answers

Answer:

x = 17/5

y = -2/5

Step-by-step explanation:

2x + 2y = 6

3x - 2y = 11

sum both equations results

5x + 0 = 17

x = 17/5

2x + 2y = 6

2*17/5 + 2y = 6

34/5 + 2y = 6

2y = 6 - 34/5

2y = 30/5 - 34/5

2y = -4/5

y = (-4/5)/2

y = -2/5

verify:

3x - 2y = 11

3*17/5 - 2*-2/5 = 11

51/5 + 4/5 = 55/5

51 + 4 = 55

(SAT Prep) Find the value of x.

Answers

Answer:

The value of x is 30°

Step-by-step explanation:

We are given that the outer angle of the parallelogram is 60 degrees. Therefore it's respective inner angle will be 180 - 60 = 120 degrees. And, by properties of a parallelogram, the angle opposite to this angle will be 120 degrees as well.

If we draw extend the line creating angle 2x, then we will make ( 1 ) a vertical angle to 2x, ( 2 ) a 90 degree angle, and ( 3 ) and angle that we can let be y. Therefore, 2x + y = 90, and 3x + y = 120.

[tex]\begin{bmatrix}2x+y=90\\ 3x+y=120\end{bmatrix}[/tex] ,

[tex]\begin{bmatrix}6x+3y=270\\ 6x+2y=240\end{bmatrix}[/tex] ,

[tex]6x+2y=240\\-\\\underline{6x+3y=270}\\y=30[/tex],  

[tex]2x + (30) = 90,\\2x = 60,\\x = 30[/tex]

Solution : x = 30°

Answer:

x = 30

Step-by-step explanation:

a+ 60 = 180

a = 120

3x+b = 120  because opposite angles in a parallelogram are equal

2x+90+b = 180 since it forms a line

2x+b = 90

We have 2 equations and 2 unknowns

3x+b = 120

2x+b = 90

Subtracting

3x+b = 120

-2x-b = -90

---------------------

x = 30

The table shows the probability distribution of student ages in a high school
with 1500 students. What is the expected value for the age of a randomly
chosen student?
Age
13
14
15
16
17
18
Probability 0.01 0.23 0.26 0.28 0.20 0.02

Answers

Answer:

Exoected age is 15.49 years

Step-by-step explanation:

Expected age

= E(x)

= sum (p(i)*i)

= 13*0.01+14*0.23+15*0.26+16*0.28+17*0.20+18*0.02

= 15.49

HELP PLEASE 50 POINTS
A mechanic charges $125 plus $25 per hour of labor. The equation c=25n+125
describes the total she would charge for a service visit, where n represents the number of
hours of labor and c is the total cost. Graph the equation and using the graph, find how
much he charges when she works 3 hours.

Answers

Answer:

See Attachment for graph

Charges = $200 when number of she works for 3 hours

Step-by-step explanation:

Given

Charges = $25 per hour + $125 (c = 25n + 125)

Required

Graph the equation.

From the graph, determine c when n = 3

To plot the graph; first, we have to determine the points to use;

When n = 1

[tex]c = 25 * 1 + 125[/tex]

[tex]c = 25 + 125[/tex]

[tex]c = 150[/tex]

When n = 2

[tex]c = 25 * 2 + 125[/tex]

[tex]c = 50 + 125[/tex]

[tex]c = 175[/tex]

When n = 3

[tex]c = 25 * 3 + 125[/tex]

[tex]c = 75 + 125[/tex]

[tex]c = 200[/tex]

When n = 4

[tex]c = 25 * 4 + 125[/tex]

[tex]c = 100 + 125[/tex]

[tex]c = 225[/tex]

Plotting n on the x axis and c on the y axis; we have

c   ||   n

1    ||   150

2   ||   175

3   ||    200

4   ||    225

(See attachment)

From the attachment;

When she works for 3 hour; This implies that n = 3

And c = $200 when n = 3

See Proof

[tex]c = 25n + 125[/tex]

Substitute 3 for n

[tex]c = 25 * 3 + 125[/tex]

[tex]c = 75 + 125[/tex]

[tex]c = 200[/tex]

Nour drove from the Dead Sea up to Amman, and her altitude changed at a constant rate. When she began driving, her altitude was 400400400 meters below sea level. When she arrived in Amman 222 hours later, her altitude was 100010001000 meters above sea level. Let yyy represent Nour's altitude (in meters) relative to sea level after xxx hours.

Answers

Answer:

y = 700x - 400

Step-by-step explanation:

A negative number represents an altitude below sea level.

Beginning: -400

y = mx + b

y = mx - 400

In 2 hours the altitude was now 1000 m.

1000 m - (400 m) = 1400 m

The altitude went up 1400 m in 2 hours. The rate of change is

1400/2 m/h = 700 m/h

The rate of change is the slope.

y = 700x - 400

Answer:

The graph answer is below :)

Step-by-step explanation:

please help with this

Answers

Answer:

[tex]\sin \left(\theta \right)-\frac{1}{2}\cos \left(2\theta \rightt)+C[/tex]

Step-by-step explanation:

We are given the graph of r = cos( θ ) + sin( 2θ ) so that we are being asked to determine the integral. Remember that [tex]\:r=cos\left(\theta \right)+sin\left(2\theta \right)[/tex] can also be rewritten as [tex]\int \cos \left(\theta \right)+\sin \left(2\theta \right)d\theta \right[/tex].

Let's apply the functional rule [tex]\int f\left(x\right)\pm g\left(x\right)dx=\int f\left(x\right)dx\pm \int g\left(x\right)dx[/tex],

[tex]\int \cos \left(\theta \right)+\sin \left(2\theta \right)d\theta \right[/tex] = [tex]\int \cos \left(\theta \right)d\theta \right+\int \sin \left(2\theta \right)d\theta \right[/tex]

At the same time [tex]\int \cos \left(\theta \right)d\theta \right=\sin \left(\theta \right)[/tex] = [tex]sin( \theta \right ))[/tex], and [tex]\int \sin \left(2\theta \right)d\theta \right[/tex] = [tex]-\frac{1}{2}\cos \left(2\theta \right)[/tex]. Let's substitute,

[tex]\int \cos \left(\theta \right)d\theta \right+\int \sin \left(2\theta \right)d\theta \right[/tex] = [tex]\sin \left(\theta \right)-\frac{1}{2}\cos \left(2\theta \right)[/tex]

And adding a constant C, we receive our final solution.

[tex]\sin \left(\theta \right)-\frac{1}{2}\cos \left(2\theta \rightt)+C[/tex] - this is our integral

Find the surface area of the regular pyramid shown in the accompanying diagram. If necessary, express your answer in simplest radical form.

Answers

Answer:

The area of the pyramid is 360 unit²

Step-by-step explanation:

Given

Base Edge, a = 10

Height, h = 12

Required

Determine the surface area

The surface area of a regular pyramid is calculated as thus;

[tex]A = a^2 + 2a\sqrt{\frac{a^2}{4} + h^2}[/tex]

Substitute values for a and h

[tex]A = 10^2 + 2 * 10 * \sqrt{\frac{10^2}{4} + 12^2}[/tex]

Evaluate all squares

[tex]A = 100 + 2 * 10 * \sqrt{\frac{100}{4} + 144}[/tex]

[tex]A = 100 + 2 * 10 * \sqrt{25 + 144}[/tex]

[tex]A = 100 + 2 * 10 * \sqrt{169}[/tex]

Take positive square root of 169

[tex]A = 100 + 2 * 10 * 13[/tex]

[tex]A = 100 + 260[/tex]

[tex]A = 360[/tex]

Hence, the area of the pyramid is 360 unit²

Answer:

B.) 360 units2

Step-by-step explanation:

I got it correct on founders education

Blake bought two iced coffees from Dutch Bros. He originally had $13.50 and now has $9. Write and solve an equation to find out how much each iced
coffee cost.

Answers

Answer:

each ice coffee is $2.25

Step-by-step explanation:

13.50 - 9 = 4.50

4.50 / 2 = 2.25

What is the solution to the linear equation?
2/5 + p = 4/5 + 3/5p​

Answers

Answer:

p = 1

Step-by-step explanation:

[tex] \frac{2}{5} + p = \frac{4}{5} + \frac{3}{5} p[/tex]

Multiply through by the LCM

The LCM for the equation is 5

That's

[tex]5 \times \frac{2}{5} + 5p = 5 \times \frac{4}{5} + \frac{3}{5}p \times 5[/tex]

We have

2 + 5p = 4 + 3p

Group like terms

5p - 3p = 4 - 2

2p = 2

Divide both sides by 2

We have the final answer as

p = 1

Hope this helps you

Dada la recta L: 3x - 2y + 1 = 0, ¿cual es la pendiente de la recta L1?

Answers

Answer:

La pendiente de la recta L es [tex]\frac{3}{2}[/tex].

Step-by-step explanation:

La recta está presentada en su forma implícita, es decir, que está bajo la forma:

[tex]f(x,y) = 0[/tex]

Para determinar la pendiente de la recta, se debe transformarla a su forma explícita, cuya fórmula es:

[tex]y = m \cdot x + b[/tex]

Donde:

[tex]x[/tex] - Variable independiente, adimensional.

[tex]y[/tex] - Variable dependiente, adimensional.

[tex]m[/tex] - Pendiente, adimensional.

[tex]b[/tex] - Intercepto, adimensional.

Entonces:

[tex]3\cdot x - 2\cdot y + 1 = 0[/tex]

[tex]2\cdot y = 3\cdot x +1[/tex]

[tex]y = \frac{3}{2}\cdot x + \frac{1}{2}[/tex]

Por simple inspección, se determina que la pendiente de la recta L es [tex]\frac{3}{2}[/tex].

Suppose your weekly local lottery has a winning chance of 1/106. You buy lottery from them for x weeks in a row. What is the probability that you never win?

Answers

Answer:

The probability mass function  that you never win [tex]^xC_o[/tex] = [tex](\dfrac{999999}{1000000})^x[/tex]

Step-by-step explanation:

Given that;

the winning chance of a weekly local lottery = [tex]\dfrac{1}{10^6}[/tex]

= [tex]\dfrac{1}{1000000}[/tex]

The probability of losing = 1 - probability of winning (winning chance)

The probability of losing = [tex]1- \dfrac{1}{1000000}[/tex]

The probability of losing =[tex]\dfrac{999999}{1000000}[/tex]

The probability mass function  that you never win [tex]^xC_o[/tex] = [tex](\dfrac{1}{10^6} )^0 ( \dfrac{999999}{1000000})^x[/tex]

The probability mass function  that you never win [tex]^xC_o[/tex] = [tex](\dfrac{999999}{1000000})^x[/tex]

Hi people, Please if someone can give me a hand, l already have done the first part of the exercise, but l cant make Angle CAB= X^0 c) calculate the lower bound for the value of tan X^0 there is the answer 1.02 (2dp) but l have no clue how to get it thanks i used Toa (Tan = Opp/ adj) but l couldnt find it thanks

Answers

Answer:

  (c) 1.02

Step-by-step explanation:

(c) The tangent is the ratio of Opposite to Adjacent. Its lowest value will be found where Opposite is lowest, and Adjacent is highest:

  min tan(A) = (min BC)/(max AB) = 75/73.5 ≈ 1.020408...(42-digit repeat)

Hellllppp!!!! Please!Match the numbers with the correct label.

Answers

Answer:

(a = 1/7 (b = .2 (c = 3/9

Step-by-step explanation:

1/7 = .14

1/4 = .25

3/9 = .33

a & b are lower than 1/4 and c is higher

a Find the amount compounded annually on Rs 25,000 for 2 years if the rates of
interest for two years ore 10 % and 12 % respectively,​

Answers

Answer:

Amount = Rs. 30250 when Rate = 10%

Amount = Rs. 31360 when Rate = 12%

Step-by-step explanation:

Given

[tex]Principal, P = Rs.\ 25,000[/tex]

[tex]Time, t = 2\ years[/tex]

[tex]Rate; R_1 = 10\%[/tex]

[tex]Rate; R_2 = 12\%[/tex]

Number of times (n) = Annually

[tex]n = 1[/tex]

Required

Determine the Amount for both Rates

Amount (A) is calculated by:

[tex]A = P(1 + \frac{r}{n})^{nt}[/tex]

When Rate = 10%, we have:

Substitute 25,000 for P; 2 for t; 1 for n and 10% for r

[tex]A = 25000 * (1 + \frac{10\%}{1})^{1 * 2}[/tex]

[tex]A = 25000 * (1 + \frac{10\%}{1})^{2}[/tex]

[tex]A = 25000 * (1 + 10\%)^{2}[/tex]

Convert 10% to decimal

[tex]A = 25000 * (1 + 0.10)^{2}[/tex]

[tex]A = 25000 * (1.10)^{2}[/tex]

[tex]A = 25000 * 1.21[/tex]

[tex]A = 30250[/tex]

Hence;

Amount = Rs. 30250 when Rate = 10%

When Rate = 12%, we have:

Substitute 25,000 for P; 2 for t; 1 for n and 10% for r

[tex]A = 25000 * (1 + \frac{12\%}{1})^{1 * 2}[/tex]

[tex]A = 25000 * (1 + \frac{12\%}{1})^{2}[/tex]

[tex]A = 25000 * (1 + 12\%)^{2}[/tex]

Convert 12% to decimal

[tex]A = 25000 * (1 + 0.12)^{2}[/tex]

[tex]A = 25000 * (1.12)^{2}[/tex]

[tex]A = 25000 * 1.2544[/tex]

[tex]A = 31360[/tex]

Hence;

Amount = Rs. 31360 when Rate = 12%

What is the equation of the line of best fit for the following data? Round the
slope and y-intercept of the line to three decimal places.

Answers

Answer:

the line of best fit can be approximated to:

y = -1.560 x + 22.105

Step-by-step explanation:

You are most likely expected to use a graphing tool are statistical program to calculate this. So enter the list of x-values separate from the list of y values and run the tool in linear regression mode.

Look at the attached image with the actual results including the line of best fit.

The equation can be written (rounding slope and y-intercept to 3 decimals) as:

y = -1.560 x + 22.105

Ava placed the point of her pencil on the origin of a regular coordinate plane. She marked a point after moving her pencil 4 units to the left and 7 units up. Which ordered pair identifies where Ava marked her point?

Answers

[tex] \Large{ \boxed{ \bold{ \color{lightgreen}{Solution:}}}}[/tex]

So, Let's solve this question by using cartesian plane.

Here, Origin is shown by (0, 0)Ava moves 4 units left from origin. On the left side of origin, negative x axis begins. So, she reached (-4, 0) now.Then, from that point she moved 7 units upwards. On the upper side, there is positive y axis. So, Finally she will reach point (-4, 7).(-4, 7) is the coordinate of point which is 4 units left from y axis and 7 units up from x axis.It lies on the second quadrant.

Well, What is cartesian plane?

A - A Cartesian coordinate system is a coordinate system that specifies each point uniquely in a plane by a set of numerical coordinates, which are the signed distances to the point from two fixed perpendicular oriented lines, measured in the same unit of length. 

━━━━━━━━━━━━━━━━━━━━

Other Questions
Does anyone know how I can fix this? I dont know what is wrong. When I click retry it restarts my laptop and returns back to this. Please help! Crosscutting typically:__________.a) creates a less restricted knowledge of narrative events than other types of editing do. b) demands a great deal of matching on action. c) provides a greater degree of continuity than shot/reverse shot does. d) creates a more restricted knowledge of narrative events than other types of editing do. 1A-MATHDiviwuetalls2 Exercise 7:Your ansvType youThe three angles in a triangle are labeled A, B and C.A = 2xB= 3xC=4xPrivate coi1) calculate X2) find the actual angle of B3) find the exterior angle to B Consider the following chromosomes and if they are affected by hemophilia. X = unaffected X chromosome, x = X chromosome affected by hemophilia, and Y = Y chromosome. If an Xx female and XY male have children, what fraction of their offspring will have an affected chromosome, and what fraction is likely to be affected by hemophilia?(1 point)A. 1/4 and 1/2B. 1/2 and 1/4C. 1/2 and 1/3D. 1/4 and 1/4I've been stuck on this question for a while, may someone please assist? Msr walkers class set up an online fund with a goal to raise 1280 3x + 5y = 8 -9x - 15y = - 24 ayudeneme con esta ecuacion de igualacion All the following statements concerning term insurance are correct EXCEPT: Group of answer choices Most policies can be renewed for additional periods without evidence of insurability. Most policies can be converted to a permanent life insurance policy. Most policies have a nonforfeiture benefit to refund a portion of a policy's cash value when coverage stops. The insurance provides protection for a certain period of time. The t distribution approaches the _______________ distribution as the sample size ___________. Professor Sanders wants to study how constant distractions during an exam affect student performance. She randomly assigns her students to different classrooms where the experimental group will receive a predetermined number of distractions while taking an exam. In this study, the independent variable is the ________ and the dependent variable is the ________. Im really confused and select all that apply questions scare me. what is the difference between skill and strategy ? (needs details) Last year, you purchased a stock at a price of $78.00 a share. Over the course of the year, you received $2.70 per share in dividends and inflation averaged 3.2 percent. Today, you sold your shares for $82.20 a share. What is your approximate real rate of return on this investment? Need help pls will give you a good rating. You're about to buy a new car for $10,000. The dealer offers you a one-year loan where you pay $860.66 every month for the next 12 months. Since you pay $860.66 * 12 = $10,328 in total, the dealer claims that the loan's annual interest rate is (10,328-10,000)/10,000 = 3.28%. What is the actual effective annual rate? What effect did the Han Dynastys conquering of Korea have on the religion in the region? A) It introduced Confucianism and Buddhism. B) It introduced Confucianism and Shintism. C) It introduced Buddhism and Shintism. D) It introduced Shintism and Christianity 1. Which expression is equivalent to (-2)(a + 6)? Two mutually exclusive projects have an initial cost of $60,000 each. Project A produces cash inflows of $30,000, $37,000, and $20,000 for Years 1 through 3, respectively. Project B produces cash inflow of $80,000 in Year 2 only. The required rate of return is 10 percent for Project A and 11 percent for Project B. Which project(s) should be accepted and why Please Answer For Brainliest!!! The ______ rate of interest is the actual rate charged by the supplier and paid by the demander of fund The hanger image below represents a balanced equation. Find the value of r that makes the equation true.